¿Se puede obtener un proceso de Wiener como el límite de un modelo de "tiempo de colisión sin memoria"?

Dejar ( norte t ) t 0 ser un proceso de intensidad de Poisson 1 , y para cada λ > 0 y t 0 dejar

W t ( λ ) = λ 0 t ( 1 ) norte λ s d s = 1 λ 0 λ t ( 1 ) norte s d s .

¿Es el caso de que para cada T > 0 , como λ la ley de la C ( [ 0 , T ] , R ) variable aleatoria valorada ( W t ( λ ) ) t [ 0 , T ] converge débilmente a la medida de Wiener en C ( [ 0 , T ] , R ) ?

(Aquí, C ( [ 0 , T ] , R ) está equipado con la topología de convergencia uniforme.)

Observación. Si la respuesta es , entonces esta puede ser una forma más intuitiva físicamente de pensar en los procesos de Wiener que como el límite de una simple caminata aleatoria: un modelo de tiempos de colisión aleatorios de ida y vuelta "se siente más motivado físicamente" (particularmente cuando tratando de visualizar el movimiento browniano físico de las partículas) que una decisión aleatoria de moverse hacia la izquierda o hacia la derecha en cada paso de tiempo de una duración fija aparentemente arbitraria. También sería una buena manera de formalizar la noción de que el ruido blanco gaussiano unidimensional ("ilimitado") se puede obtener como un límite del ruido de Markov dicotómico ("limitado").

Si la respuesta a la pregunta es , entonces esto parece un hecho muy básico; ¿Hay alguna referencia con este hecho (ya sea como un teorema o un ejercicio)?


Mi muy cruda intuición para una respuesta positiva:

Se "siente obvio" que para grandes λ , el proceso estocástico ( W t ( λ ) ) t 0 tiene incrementos independientes "aproximadamente" estacionarios, y mi [ W t ( λ ) ] 0 para todos t 0 .

Así que ahora, para fijo τ > 0 , consideremos la forma y la varianza de la distribución de W τ ( λ ) . Para cada norte norte , dejar T norte = inf { t > 0 : norte λ t = norte } . Dado que las variables aleatorias T i T i 1 son independientes y exponencialmente distribuidos con varianza 1 λ 2 , la varianza de W T norte ( λ ) es norte λ , y aplicando el teorema del límite central a T 1 + ( T 3 T 2 ) + ( T 5 T 4 ) + y para ( T 2 T 1 ) + ( T 4 T 3 ) + ( T 6 T 5 ) + da eso por grande norte , la distribución de W T norte ( λ ) tiene una forma aproximadamente normal. De ahí la distribución de Y := W T λ τ ( λ ) es aproximadamente una distribución normal con una varianza de aproximadamente τ . Ahora si arreglamos un pequeño ε > 0 , escribiendo I ε para el intervalo estocástico

I ε = [ T λ ( τ ε ) , T λ ( τ + ε ) ] ,
tomando lo suficientemente grande λ debería dar eso

  • τ I ε con alta probabilidad y
  • la variable aleatoria máximo t I ε | W t ( λ ) Y | esta cerca de 0 con alta probabilidad.

Por tanto, parece intuitivo que la ley de W τ ( λ ) debe ser aproximadamente igual a la ley de Y , y así en particular, debería estar aproximadamente distribuida normalmente con una varianza aproximada τ .


Ahora publiqué una pregunta más general relacionada en MathOverflow, https://mathoverflow.net/questions/360363 . Si la respuesta a esa pregunta es , entonces la respuesta a esta también debería serlo (por el argumento de zhoraster usando el principio de invariancia generalizada preguntado en esa pregunta).

La convergencia a W sigue el mismo camino, agrupando las variables de dos en dos, solo que en lugar de CLT usa el principio de invariancia.
Se parece mucho a la caminata aleatoria de tiempo continuo: en.wikipedia.org/wiki/Continuous-time_random_walk
@ user619894 No creo que este sea un paseo aleatorio de tiempo continuo. Parte de la definición de una caminata aleatoria (en tiempo continuo o discreto) suele ser alguna noción de "incrementos de iid" (por ejemplo, como en su artículo vinculado de Wikipedia). Por el contrario, el proceso que estoy describiendo alterna de forma determinista entre moverse a la izquierda y moverse a la derecha, solo que con tiempos de espera aleatorios.
@zhoraster Vamos Δ T norte ser el norte -th tiempo de espera para el proceso de Poisson ( norte λ t ) . Si estuviera mirando el proceso cuyo valor en el tiempo 2 norte λ es λ ( Δ T 1 Δ T 2 + Δ T 3 Δ T 4 + Δ T 2 norte ) , entonces puedo ver que la IP daría el resultado deseado. Pero el punto es que estoy viendo el proceso cuyo valor es λ ( Δ T 1 Δ T 2 + Δ T 3 Δ T 4 + Δ T 2 norte ) no a la hora 2 norte λ sino más bien a tiempo Δ T 1 + Δ T 2 + Δ T 3 + Δ T 4 + + Δ T 2 norte .
Tienes razón. Pero, gracias al SLLN, Δ T 1 + Δ T 2 + Δ T 3 + Δ T 4 + + Δ T 2 norte 2 norte λ , así que esto no importa en absoluto.
@zhoraster Después de ver su comentario original, también estaba pensando que algún argumento en este sentido podría solucionar el problema, pero estoy bastante seguro de que necesitamos algo más que LLN. Si tuviéramos algo como λ máximo 1 norte 2 λ | Δ T 1 + + Δ T 2 norte 2 norte λ | 0 en distribución como λ , entonces esto sería suficiente, pero desafortunadamente, estoy bastante seguro de que CLT nos dice que no lo hacemos.
Escribiste algo raro: cuando λ > 2 , tienes máximo 1 norte 0 .
@zhoraster Lo siento, el subíndice pretende decir 1 norte λ 2 . (En realidad, podría reemplazar λ 2 con C λ para cualquier constante C . Si estamos pensando en la convergencia en [ 0 , 1 ] , entonces básicamente queremos C = 1 2 .)
¿Y por qué necesitas máximo por el principio de invariancia?
@zhoraster En la recomendación automática de SE, ahora moví la discusión a "chat" (vea el comentario anterior para el enlace).
@zhoraster Gracias por su ayuda, sobre la base de la cual ahora he escrito una respuesta completa a la pregunta.

Respuestas (1)

Con la ayuda de MathOverflow, en https://mathoverflow.net/questions/360363/ , ahora puedo convertir la sugerencia de zhoraster en una respuesta completa:

Definir el proceso estocástico ( W ~ t ( λ ) ) t 0 tal que W ~ t ( λ ) está de acuerdo con W t ( λ ) en cada segundo evento del proceso de Poisson ( norte λ t ) t 0 y se interpola linealmente en el medio. Es decir,

W ~ 1 λ ( r S 2 norte + ( 1 r ) S 2 norte + 2 ) ( λ )   =   r W 1 λ S 2 norte ( λ )   +   ( 1 r ) W 1 λ S 2 norte + 2 ( λ ) para todos  norte 0 , r [ 0 , 1 ]
dónde
S norte := inf { t 0 : norte t = norte } .
Dejar D norte = S norte + 1 S norte . Podemos aplicar el principio de invariancia de Donsker generalizado en la pregunta MO vinculada, con Δ norte = D 2 norte + D 2 norte + 1 y X norte = D 2 norte D 2 norte + 1 , para producir que en cualquier intervalo de tiempo compacto el proceso W ~ t ( λ ) converge en distribución (wrt la topología de convergencia uniforme) al proceso de Wiener.

Así que solo queda controlar la diferencia W ~ t ( λ ) W t ( λ ) . Para cualquier T > 0 ,

máximo t [ 0 , T ] | W ~ t ( λ ) W t ( λ ) |     máximo { D norte : 0 norte norte λ T + 1 } λ .
Ahora para cualquier ε > 0 , tomando suficientemente grande λ dará eso PAG ( norte λ T + 1 2 λ T ) > 1 ε 2 ; y desde
máximo { D norte : 0 norte 2 λ T } registro ( 2 λ T )
es convergente en distribución como λ (a la distribución Gumbel) y registro ( X ) X 0 como X , se sigue que para suficientemente grande λ ,
PAG ( máximo { D norte : 0 norte 2 λ T } λ > ε )   <   ε 2
y por lo tanto
PAG ( máximo { D norte : 0 norte norte λ T + 1 } λ > ε )   <   ε .
Por eso máximo t [ 0 , T ] | W ~ t ( λ ) W t ( λ ) | converge en probabilidad a 0 , por lo que se sigue que ( W t ( λ ) ) t [ 0 , T ] converge en distribución al proceso de Wiener en [ 0 , T ] .